Verbal problems from the *free* official practice tests and
problems from mba.com
az780
 
 

cr (Industrialists)

by az780 Fri Sep 12, 2008 5:32 pm

Industrialists from the country Distopia were accused of promoting the Distopian intervention in the Arcadian civil war merely to insure that the industrialists’ facilities in Arcadia made substantial profits during the war. Yet this cannot be the motive since, as the Distopians foresaw, Distopia’s federal expenses for the intervention were eight billion dollars, whereas, during the war, profits from the Distopian industrialists’ facilities in Arcadia totaled only four billion dollars.
Which of the following, if true, exposes a serious flaw in the argument made in the second sentence above?
(A) During the Arcadian war, many Distopian industrialists with facilities located in Arcadia experienced a significant rise in productivity in their facilities located in Distopia.
(B) The largest proportion of Distopia’s federal expenses is borne by those who receive no significant industrial profits.
(C) Most Distopian industrialists’ facilities located in Arcadia are expected to maintain the level of profits they achieved during the war.
(D) Distopian industrialists’ facilities in Arcadia made substantial profits before the events that triggered the civil war.
(E) Many Distopians expressed concern over the suffering that Arcadians underwent during the civil war.
Guest
 
 

by Guest Fri Sep 12, 2008 9:35 pm

A) During the Arcadian war, many Distopian industrialists with facilities located in Arcadia experienced a significant rise in productivity in their facilities located in Distopia. -- There is no reason to believe that this rise will not go back to normal once the war is over.

(B) The largest proportion of Distopia’s federal expenses is borne by those who receive no significant industrial profits. -- no relevance

(C) Most Distopian industrialists’ facilities located in Arcadia are expected to maintain the level of profits they achieved during the war. --If they continue to maintain the levels of profits then they are bound to exceed $8B down the line

(D) Distopian industrialists’ facilities in Arcadia made substantial profits before the events that triggered the civil war. -- this would certainly provide an alternate cause therefore strengthening the authors argument.

(E) Many Distopians expressed concern over the suffering that Arcadians underwent during the civil war. -- no relevance.

"Yet this cannot be the motive since" is the conclusion; therefore C is the correct answer as it shows a flaw in the arguement provided to reach the conclusion.
Guest
 
 

by Guest Sat Sep 20, 2008 8:08 am

can someone explain this plz :(
Priyanka
 
 

by Priyanka Tue Sep 23, 2008 6:09 am

Wht is asked is a flaw in the reasoning ,

The stimulus tries to prove that the motive for civil war to generate Profit for D's businesses in A. But since the federal expenses were 8 billion dollars and the profit generated by the Industries was only 4 billion $. The motive could not have been to increase war.

We have to find an answer choice that negates what is said and proves that the utimate reason for war was to generate Profits.

Ans choice (B) by saying that the The Federal Expenses were borne by those other than the industrialists proves that the point.

Hence (B) is the best answer.
RonPurewal
Students
 
Posts: 19744
Joined: Tue Aug 14, 2007 8:23 am
 

by RonPurewal Wed Oct 22, 2008 7:00 am

yeah, so there's a rather large logic gap in this one. specifically:
* the profit of $4b went to the industrialists.
* the cost of $8b was borne by the feds.
the argument assumes - completely without justification - that the $8b cost to the feds will somehow cancel out the industrialists' profit. there's no reason that this should be the case, or, for that matter, that the costs and revenues of those two entities should have anything to do with one another.
therefore, to expose that flaw, we need an answer choice that demonstrates that the feds' and industrialists' ledgers are independent of each other, at least to a large enough degree that the industrialists can still make a tidy profit.
choice (b) does this.
sangeethmani
Students
 
Posts: 22
Joined: Tue Oct 27, 2009 8:03 am
 

Re: cr (Industrialists). Please help

by sangeethmani Thu Aug 12, 2010 6:00 pm

I am not convinced with the answer for one simple question. As the "as the Distopians foresaw, Distopia's federal expenses for the intervention were eight billion dollars, whereas, during the war, profits from the Distopian industrialists' facilities in Arcadia totaled only four billion dollars. "

If the industrialists foresaw that they would make 4 billion out of the 8 billion they invest , how can they anticipate profits?

The answer choice (b) says that "The largest proportion of Distopia’s federal expenses is borne by those who receive no
significant industrial profits". If this is true then why would the industrialists agree to invest in the 8 billion at all is they know they are going to make a loss. Hence it does not weaken the argument that they did not do this for profits. It infact strengthens the argument to an extent that the did not anticipate any profits in this venture.

Could I please get an explanation on this one.

For me C seems to be right.
mschwrtz
ManhattanGMAT Staff
 
Posts: 498
Joined: Tue Dec 14, 2004 1:03 pm
 

Re: cr (Industrialists)

by mschwrtz Fri Sep 03, 2010 2:58 pm

the industrialists did not foresee "that they would make 4 billion out of the 8 billion they invest." They foresaw that the government would spend 8 billion, and the industrialists would make 4 billion.
thanghnvn
Prospective Students
 
Posts: 711
Joined: Wed Jan 14, 2009 9:09 pm
 

Re: cr (Industrialists)

by thanghnvn Fri Mar 16, 2012 11:54 pm

Ron, Tim, pls, help.

I find an assumption that profit before the war is larger than in the war. I choose d and wrong.

If d said: the profit of the industries before the war is LARGER than that in the war, d is correct.

d said that the profit of the industries is substantial

is my thinking correct?
RonPurewal
Students
 
Posts: 19744
Joined: Tue Aug 14, 2007 8:23 am
 

Re: cr (Industrialists)

by RonPurewal Wed Apr 04, 2012 5:28 pm

thanghnvn Wrote:Ron, Tim, pls, help.

I find an assumption that profit before the war is larger than in the war. I choose d and wrong.

If d said: the profit of the industries before the war is LARGER than that in the war, d is correct.

d said that the profit of the industries is substantial

is my thinking correct?



nope. the argument is concerned only with the dollars and cents of industrial activity during the war, not before it. whatever transpired before the war is not relevant to the argument actually contained in the passage.
shans.bgp
Students
 
Posts: 14
Joined: Sat May 12, 2012 5:49 am
 

Re: cr (Industrialists)

by shans.bgp Sat Sep 15, 2012 8:37 am

I think the conclusion is "Yet this cannot be the motive ". So making profits is not the motive of D's intervention in A. The option B Strengthens the conclusion by saying that yes the companies supporting Fed are not the ones who make profit.

Second, I know we have to make explanations to suit OAs, yet i want to know why C is wrong. C weakens by saying that yes profit is the motive because they will recover with profits in the coming years.

Please help.
RonPurewal
Students
 
Posts: 19744
Joined: Tue Aug 14, 2007 8:23 am
 

Re: cr (Industrialists)

by RonPurewal Mon Sep 24, 2012 8:36 am

shans.bgp Wrote:I think the conclusion is "Yet this cannot be the motive ". So making profits is not the motive of D's intervention in A. The option B Strengthens the conclusion by saying that yes the companies supporting Fed are not the ones who make profit.


you've got this backward -- read the passage again, and make sure you are crystal clear on what is actually its point. (yes, this seems to be one of the harder passages to understand; personally, i had to read it 2-3 times.)

the passage is trying to make the point that the industrialists supported the war for reasons other than profit. to substantiate this point, the argument notes that the federal government spent more money than the industrialists took in.
when the argument is put in those terms, the weakness becomes a lot more glaring -- basically, you should think, "well, why would the industrialists care about the federal government's balance sheet?"

the correct answer demonstrates that the industrialists actually don't care about the federal government's balance sheet, and so weakens the argument.


choice (c) doesn't relate to the argument made in the second sentence.
gmatwork
Course Students
 
Posts: 185
Joined: Wed Dec 31, 1969 8:00 pm
 

Re: cr (Industrialists)

by gmatwork Mon Oct 29, 2012 1:50 pm

can someone please explain how to eliminate choice a?
jlucero
Forum Guests
 
Posts: 1102
Joined: Wed May 12, 2010 1:33 am
 

Re: cr (Industrialists)

by jlucero Fri Nov 09, 2012 5:00 pm

erpriyankabishnoi Wrote:can someone please explain how to eliminate choice a?


The question is asking us to expose a flaw in the argument. Not to find things that would strengthen or weaken the case for promoting intervention in the war. In Ron's words:

* the profit of $4b went to the industrialists.
* the cost of $8b was borne by the feds.

If you had a chance to make $4b, you would gladly take that money, even if you were making more money than usual elsewhere while the war was happening (A). When you wouldn't want to make $4b is if you had to spend $8b to do so (B).
Joe Lucero
Manhattan GMAT Instructor
AsadA969
Course Students
 
Posts: 312
Joined: Tue Sep 24, 2024 2:38 pm
 

Re: cr (Industrialists)

by AsadA969 Sun Apr 26, 2015 6:25 am

Government expense is 8 billions while profit is 4 billions . So loss is 4 billion . Here author assumes that industrialists bear the cost of war . B has perfectly attacked this assumption . As industrialist does not bear cost of war , then they still can lobby government to intervene at war . No matter cost of war is higher than profit . So, B is the correct one.
RonPurewal
Students
 
Posts: 19744
Joined: Tue Aug 14, 2007 8:23 am
 

Re: cr (Industrialists)

by RonPurewal Sun Apr 26, 2015 1:03 pm

^^ do you have a question? please clarify, thanks.